Question

Q2. Fig Q2 shows the block diagram of an unstable system with transfer function G(s) - under the control of a lead compensator (a) Using the Rouths stability criterion, determine the conditions on k and a so that the closed-loop system is stable, and sketch the region on the (k, a)- plane where the conditions are satisfied. Hence, determine the minimum value of k for the lead compensator to be a feasible stabilizing controller. (10 marks) (b) Suppose α-2. Given that-K(s)G(s)-0 has three roots at s ds 0.218, -2.61+j1.53, sketch the root locus diagram for 0<k< oo. Hence determine (10 marks) the range of k for which the closed-loop system is stable. Fig. Q2

0 0
Add a comment Improve this question Transcribed image text
Answer #1

Answes !- The open Leop tranter bunction for Gtven Sysem equation of the Syskmi Solving the above ean Now Forming Rouths ата. S 4K-22-ko+10 for lesed loop Sys tem to be stablte, The first celocom olColu mnet Routh arsay does not contain zero as element うー% >10Figure 1

Figure 2

Add a comment
Know the answer?
Add Answer to:
Q2. Fig Q2 shows the block diagram of an unstable system with transfer function G(s) -...
Your Answer:

Post as a guest

Your Name:

What's your source?

Earn Coins

Coins can be redeemed for fabulous gifts.

Not the answer you're looking for? Ask your own homework help question. Our experts will answer your question WITHIN MINUTES for Free.
Similar Homework Help Questions
  • Q. 1 (5 marks) For the system in Fig. (a). Assume proportion control, Gc(s)-K, sketch the root lo...

    pls answer dont just copy other solution or ur catching a dislike Q. 1 (5 marks) For the system in Fig. (a). Assume proportion control, Gc(s)-K, sketch the root locus for the closed-loop system (b). Using the angle condition, prove that s12 +j2 is not on the root locus. (c). Design a lead compensator Ge(s) - K such that the dominant closed-loop poles are located at s1--2 2. (d), What are the zero and pole of lead compensator G() (e)....

  • Q. 1 (10 marks) For the system in Fig. 1 (a) Assume proportion control. Ge(s) =...

    Q. 1 (10 marks) For the system in Fig. 1 (a) Assume proportion control. Ge(s) = K. sketch the root locus for the closed-loop system (b). Using the angle condition, prove that s1 =-2 +j2 is not on the root locus. (c). Design a lead compensator such that the dominant closed-loop poles are located at s-2tj2. (d). What are the zero and pole of lead compensator Ge(s)? (e). With Ge (s) has the zero and pole found in (c), sketch...

  • QUESTION a) With appropriate diagram explain the following compensator arrangements i. Cascade ii. Minor Loop Feedback...

    QUESTION a) With appropriate diagram explain the following compensator arrangements i. Cascade ii. Minor Loop Feedback iii. Output Feedback (6 marks) b) Design a suitable lead compensator for a system with unity feedback and having open loop transfer function: Gis) = $(5+10) Determine the gain k meet the following specifications: Damping ratio { = 0.5 b. Undamped natural frequency w Hence determine the settling time, peak overshoot and peak time for a unit step input (10 marks). c) Sketch the...

  • Q1. Show analytically that the Root Locus for the unity feedback system with open loop transfer f...

    Q1. Show analytically that the Root Locus for the unity feedback system with open loop transfer function: (a) [10 marks] K(s 4) (s + 2) is a circle, and find the centre and the radius. Determine the minimum value of the damping ratio and the corresponding value of K (b) The root locus of the open loop transfer function: [10 marks] s(s26s +15) is depicted in Figure Q1(b). Find the minimum value of gain K that will render the system...

  • Q.2 (10 marks) Consider the system shown in Fig.2 with K(5-3) H(s) = (s – 4)...

    Q.2 (10 marks) Consider the system shown in Fig.2 with K(5-3) H(s) = (s – 4) (s+1)(s+2) (a) Sketch the root locus of the closed-loop system as the gain K varies from zero to infinity. (b) Based on the root locus, determine the range of K such that the system is stable and under-damped. (c) Determine the K value such that the closed-loop system is over-damped and stable. (d) Use MATLAB draw the root locus and confirm the root locus...

  • Design Problems: (1) A robotic system is described by the transfer function P(s)=- 100 s(s +9.7)(s...

    Design Problems: (1) A robotic system is described by the transfer function P(s)=- 100 s(s +9.7)(s + 51.2) Use the root locus method to design a lead controller that achieves a closed-loop step response with P.0.5 2.5 %, and a settling time T, < 0.25s (using the 2% criterion). Also, the steady-state error to a unit ramp should be ess < 0.15. (2) This system is open-loop unstable: P(S) = 500 (5 - 1)(s + 10) Using the root locus...

  • Consider the system shown as below. Draw a Bode diagram of the open-loop transfer function G(s).

    1 Consider the system shown as below. Draw a Bode diagram of the open-loop transfer function G(s). Determine the phase margin, gain-crossover frequency, gain margin and phase-crossover frequency, (Sketch the bode diagram by hand) 2 Consider the system shown as below. Use MATLAB to draw a bode diagram of the open-loop transfer function G(s). Show the gain-crossover frequency and phase-crossover frequency in the Bode diagram and determine the phase margin and gain margin. 3. Consider the system shown as below. Design a...

  • Question 1 (60 points) Consider the following block diagram where G(s)- Controller R(s) G(s) (a) Sketch the root locus assuming a proportional controller is used. [25 points] (b) Design specifica...

    Question 1 (60 points) Consider the following block diagram where G(s)- Controller R(s) G(s) (a) Sketch the root locus assuming a proportional controller is used. [25 points] (b) Design specifications require a closed-loop pole at (-3+j1). Design a lead compensator to make sure the root locus goes through this point. For the design, pick the pole of the compensator at-23 and analytically find its zero. (Hint: Lead compensator transfer function will be Ge (s)$+23 First plot the poles and zeros...

  • 4. A lead compensator with a transfer function Ge(s)=K(+0.5/(s+3) has been designed for a Space vehicle...

    4. A lead compensator with a transfer function Ge(s)=K(+0.5/(s+3) has been designed for a Space vehicle with the transfer function 1/s' such that at the dominant closed loop poles are located at -1 +/-j1. (0) What is the angle deficiency of the uncompensated system at the designed point provided by the location of the dominant poles? Show that the compensator provides the necessary lead angle at the designed point to satisfy the root locus angle criterion. What value of K...

  • 4. A lead compensator with a transfer function Ge(s) = K(s+0.5)/(s+3) has been designed for a...

    4. A lead compensator with a transfer function Ge(s) = K(s+0.5)/(s+3) has been designed for a Space vehicle with the transfer function 1/s? such that at the dominant closed loop poles are located at -1 +/-jl. (1) What is the angle deficiency of the uncompensated system at the designed point provided by the location of the dominant poles? Show that the compensator provides the necessary lead angle at the designed point to satisfy the root locus angle criterion. (iii) What...

ADVERTISEMENT
Free Homework Help App
Download From Google Play
Scan Your Homework
to Get Instant Free Answers
Need Online Homework Help?
Ask a Question
Get Answers For Free
Most questions answered within 3 hours.
ADVERTISEMENT
ADVERTISEMENT
ADVERTISEMENT